LSAT and Law School Admissions Forum

Get expert LSAT preparation and law school admissions advice from PowerScore Test Preparation.

 Administrator
PowerScore Staff
  • PowerScore Staff
  • Posts: 8917
  • Joined: Feb 02, 2011
|
#72934
Complete Question Explanation

Strengthen-Principle. The correct answer choice is (E).

There is no argument in this stimulus, only a fact pattern that describes what a salesperson did: recommended to a landlord a less powerful air conditioning unit that was the same price as a more powerful one, on the grounds that the recommended unit was powerful enough. At first glance this might seem like bad advice. Why not get the more powerful unit if it is the same price? Why settle for good enough when something better is available at no extra charge?

The stem asks us to identify a principle (a rule, a guideline) that would help us to understand why the salesperson did what they did. When we are asked about a stimulus "conforming" to a principle, we mean that the fact pattern or argument followed the rule in the correct answer, and that the rule in that answer supports, explains, or otherwise helps make sense of the stimulus.

So what rule would help make our salesperson look good for doing what they did? A rule that says a salesperson should always recommend the least powerful unit that is good enough for the job, or that says a customer should never buy something more powerful than they need.

Answer choice (A): This answer tells us nothing about what either the salesperson of the customer should do in these circumstances, only what could be satisfactory. We need a rule about what these people should have done, so this is a loser.

Answer choice (B): An opposite answer here, which suggests that the customer should have in this case bought the MORE powerful unit! This would undermine, rather than support, the actions of the salesperson.

Answer choice (C): While this answer at least gets to what the salesperson should do, and might be somewhat attractive as a result, it muddies the waters a bit by bringing up the idea of "best value." What's a better value - a more powerful unit, or a less powerful one for the same price? The stimulus doesn't give us enough information to know what the better value is. Perhaps the less powerful unit is a better value due to using less energy, or needing less maintenance, or lasting longer? Or maybe the more powerful unit is the better value for any of those same reasons? We simply cannot tell whether this rule helps, hurts, or has no impact on the stimulus, and so it is a loser.

Answer choice (D): Because this answer focuses on the salesperson's commission, about which we know absolutely nothing from the stimulus, it doesn't give us enough information to know whether this rule helps, hurts, or has no impact on the stimulus.

Answer choice (E):This is the correct answer choice. At last, an answer that matches our prephrase! If the customer should buy the least powerful unit that meets her needs, then the salesperson't recommendation at last makes clear sense. This rule, when applied to the stimulus, helps to explain the actions of the salesperson, and is therefore correct.
 15veries
  • Posts: 113
  • Joined: Sep 25, 2016
|
#29533
Hello,

I just wanna make sure;
So for principle/justify question, we can always go extreme right?
In the stimulus it only says "less" but E says "least."
Since principle is least, less is included in it, thus correct. Is this correct?
 Clay Cooper
PowerScore Staff
  • PowerScore Staff
  • Posts: 241
  • Joined: Jul 03, 2015
|
#29540
Hi again 15veries,

Thanks for another good question.

You can definitely 'go extreme' in this case, but I would be careful generalizing from that. Though the unit recommended by the salesperson is only said to be 'less' powerful, and the answer choice says 'least', note that since there are only two options, less does in fact mean least here.

Instead of thinking of principle/justify questions this way, I would encourage you to think of them this way: you need to find the answer choice that contains a rule that would prove your conclusion. It seems like you understand this idea from the rest of your post; I would emphasize it in my approach to these questions moving forward.

Keep working hard!
 SamHuangWLS
  • Posts: 3
  • Joined: Apr 27, 2017
|
#36368
Hi,

I have been struggling between (C) and (E).

I finally chose (C) because it is from the salesperson's standpoint, because she is giving advice and the question stem specifically asks "The SALESPERSON'S advice...". I think it is logical to look for the answer choice that consistent with the perspective of the stimulus and question stem. However, I am not so sure about "best value", because, there is no way to infer what criteria to judge the value, which could electricity fee, maintenance cost, etc.

As for (E), I ruled it out because it is from the consumer's perspective, despite the rest is good.

As I mentioned before, I personally find that the recent tests are not a bit loosing on wording. I understand that manipulation of words is the test makers' tactics, but it is frustrating to encounter questions like this. I don't know how ti tackle this "new type" of questions. Please help. Thank you.
 Luke Haqq
PowerScore Staff
  • PowerScore Staff
  • Posts: 742
  • Joined: Apr 26, 2012
|
#36464
Hi Sam,

You're certainly right to question the "best value" part in answer choice (C). There isn't anything in the stimulus that clearly states that one or the other option is the best value. At the same time, one could try to argue that talk of "best value" came up in the saleswoman's advice that the "Sno-Queen was the most powerful unit for the price." Even if that were assumed, though, it wouldn't be a principle that the salesperson's advice adhered to, since she didn't recommend the Sno-Queen but instead the FreezAll.

(E), by contrast, refers to the principle "that consumer should choose the least powerful product that meets his or her needs." We can see this principle in play in the saleswoman's advice because she recommended the FreezAll to the landlord, "saying that the FreezAll was powerful enough for his needs." The "powerful enough" language in her recommendation is why one can infer that she's recommending not the most powerful unit for the price, but rather is recommending the one that has the right amount of power to meet the consumer's needs without being more powerful than necessary--in other words, she's recommending the least powerful product that meets the consumer's needs.

Hope that helps!

Get the most out of your LSAT Prep Plus subscription.

Analyze and track your performance with our Testing and Analytics Package.